Tomaz realized that the tip of a second hand on a clock rotates about the center of the clock. He watched the second hand rotate around the center of the clock for 15 seconds. Which describes the rotation he observed?
270 degrees clockwise rotation
90 degrees clockwise rotation
180 degrees rotation
90 degrees counterclockwise rotation

Answers

Answer 1

Answer:

Step-by-step explanation:

Second hand on a clock rotates at rate 360 degrees per 60 seconds.

360 degrees - 60 sec

x degrees - 15 sec

90 clockwise rotation (none hand of a clock rotates counter-clockwise :))

Tomaz Realized That The Tip Of A Second Hand On A Clock Rotates About The Center Of The Clock. He Watched
Answer 2

Answer:

90 degrees clockwise rotation

Step-by-step explanation:


Related Questions

Which of the following is equivalent to 2/x + 3/x−1 for x>1 ?

pls explain

Answers

Answer:

Step-by-step explanation:

The perimeter of a rectangular swimming pool is 56 meters. The width is 4 meters less than the length. What is the width of the swimming pool? 4 meters 4 meters 8 meters 8 meters 12 meters 12 meters 24 meters

Answers

Answer:

Length = 16 meters

Width = 12 meters

Step-by-step explanation:

Perimeter of a rectangle = 2(length + width)

Let

length = x

Width = (x - 4) meters

Perimeter of the rectangular pool = 56 meters

Perimeter of a rectangle = 2(length + width)

56 = 2{x + (x - 4)}

56 = 2(x + x - 4)

56 = 2(2x - 4)

56 = 4x - 8

56 + 8 = 4x

64 = 4x

x = 64/4

x = 16

length = x = 16 meters

Width = (x - 4) meters

= 16 - 4

= 12 meters

I'm confused, can someone please help?!​

Answers

Answer:

B

Step-by-step explanation:

Pls brain list if right :>

Answer:

If ABCD is congruent to RSTU

AB≅RS

BC≅ST

CD≅TU

AD≅RU

and ∠A≅CR

∠B≅∠S

∠C≅T

∠D≅CU

ANSWER: ∠A≅∠U

------------------------------

hope it helps...

have a great day!!

If you know the answer to any of these please Ill mark brainliest.

Answers

Answer:

Step-by-step explanation:

1) 2/9 = 185/x

  2x = 1665

x = 832.5 or 8.325 meters

2) unit rate is the cost of one unit

i.e a dozen eggs cost $1.20 that

would be 10 cents each

3) 300 tissues for $3.75 vs. 250 for $2.99

one is 1.24 cents per tissue

the other 1.19 ... the 250 package is a better buy

Selecting a few households from New York City and observing whether or not they own stocks when it is known that 28% of all households in New York City own stocks. Is this experiment a binomial experiment? Explain why.

Answers

Answer:

Since for each household there are only two possible outcomes, the a household owning stock is independent of any other household and there is a fixed number of trials, this experiment is a binomial experiment.

Step-by-step explanation:

We have to take into consideration three things:

For each household, there are only two possible outcomes, either they own stocks, or they do not.

The probability of a household owning stock is independent of any other household, that is, for any household, the probability of someone owning stocks is 28%.

There is a fixed number of trials.

Is this experiment a binomial experiment?

Since for each household there are only two possible outcomes, the a household owning stock is independent of any other household and there is a fixed number of trials, this experiment is a binomial experiment.

A 4-column table with 4 rows. Column 1 is labeled number of friends with entries 3, 5, 7, 9. Column 2 is labeled Carnival Cost with entries 51.5, 75.5, 99.5, 123.5. Column 3 is labeled Aquarium Cost with entries 43.5, 72.5, 101.5, 130.5. Column 4 is labeled Wave Pool Cost with entries 50.25, 61.25, 85.75, 110.25.
Gale wants to compare the cost of the events.

Aquarium: $14.50 each ticket

Carnival: c = 15.5 + 12f

Wave pool: $16.75 each, but $12.25 each for groups larger than 4

Which of the claims Gale makes is true?
The table represents all possibilities.
The carnival always costs the least.
The Aquarium always costs the most.
(7, 101.5) is an ordered pair used to represent the aquarium cost.

Answers

Answer:

D.(7, 101.5) is an ordered pair used to represent the aquarium cost.

Hope it helps :)

Step-by-step explanation:

Answer: it is d - (7, 101.5) is an ordered pair used to represent the aquarium cost

Step-by-step explanation: got it right on edge 2022

Jessica is buying chicken wings and hamburger meat for a party. One bag of chicken wings costs $6. Hamburger meat costs $3 per pound. She must spend no more than $30. She also knows that she needs to buy at least 5 pounds of hamburger meat. Which system of inequalities can be used to determine the number of bags of chicken wings, x, and the number of pounds of hamburger meat, y, that Jessica should buy?(1 point)

Answers

Answer:

6x + 3y ≤ 30

y ≥ 5

Step-by-step explanation:

Let

x = number of bags of chicken wings

y = number of pounds of hamburger meat

Cost of one bag of chicken wings = $6

Cost of one pound of Hamburger meat = $3

She must spend no more than $30.

The inequality

6x + 3y ≤ 30

She also knows that she needs to buy at least 5 pounds of hamburger meat.

y ≥ 5

Solve the equation: 12 - x (x - 3) = (6 - x)(x + 2)

Answers

Answer: the answer is 0

Why 0?:

Step 1: Simplify both sides of the equation

-x * x= -x²

-x * -3= 3x all  together it is ( -x²+3x+12)

for the other side:

-x * x= -x²

6 * x= 6x and x * 2= 2x and 6 * 2= 12

then subtract 6x-4x= 2x so final would be = −x²+4x+12

Step 2: Add x² to both sides.

−x²+3x+12+x²=−x²+4x+12+x² then we are left with 3x+12=4x+12

Step 3: Subtract 4x from both sides.

3x+12−4x=4x+12−4x  --> −x+12=12

Step 4: Subtract 12 from both sides.

−x+12−12=12−12

−x=0

Step 5: Divide both sides by -1 or -x

x=0

If g(x) = 2 |x| − 1, what is g(−2.3)?

Answers

Answer:

g(-2.3) = 3.6

General Formulas and Concepts:

Pre-Algebra

Order of Operations: BPEMDAS

Brackets Parenthesis Exponents Multiplication Division Addition Subtraction Left to Right

Algebra I

FunctionsFunction Notation

Step-by-step explanation:

Step 1: Define

Identify

g(x) = 2|x| - 1

Step 2: Evaluate

Substitute in x [Function g(x)]:                                                                         g(-2.3) = 2|-2.3| - 1Absolute values:                                                                                               g(-2.3) = 2(2.3) - 1Multiply:                                                                                                             g(-2.3) = 4.6 - 1Subtract:                                                                                                            g(-2.3) = 3.6
3.6
2 *| -2.3 | - 1 = 3.6

Please help me out here

Answers

Answer:

vbw-kafw-hxy p.l.e.a.s.e join

Answer:

146 cm²

Step-by-step explanation:

The net is composed of 3 sets of congruent rectangles

top/ bottom + front/ back + sides

SA = 2(9 × 5) + 2(9 × 2) + 2(5 × 2)

     = 2(45) + 2(18) + 2(10)

     = 90 + 36 + 20

     = 146 cm²

Sam has 24 ball he gives 1/3 to Rita. How many ball did he give to Rita?​

Answers

Answer:

8

Step-by-step explanation:

Find 1/3 of 24, which is 8.

Answer:

Sam gave 8 balls to Rita.

Step-by-step explanation:

24 / 3 = 8

1/3 of 24 is 8, therefore Sam gave 8 balls to Rita.

A point P(3, k) is first transformed by E¹[0, 2] and then by E²[0,3/2] so that the final image is (9, 12), find the value of k.​​

Answers

Hello,

The first transform E1 is the homothetie of center (0,0) and ratio=2

The second transform E2 is the homothetie of center (0,0) and ratio=3/2

P=(3,k)

P'=E1(P)= E1((3,k))=(2*3,2*k)=(6,2k)

P''=E2(P')=E2(6,2k)=(3/2*6,3/2*2*k)=(9,3k)=(9,12)

==> 3k=12

k=4

Solve the equation and enter the value of x below. 7(x + 9) + 5 = 96​

Answers

Hello!

[tex]\large\boxed{x = 4}[/tex]

7(x + 9) + 5 = 96

Distribute:

7x + 63 + 5 = 96

Combine like terms:

7x + 68 = 96

Subtract 68 from both sides:

7x = 28

Divide both sides by 7:

x = 4

Answer:

[tex]\fbox{x = 7}[/tex]

Step-by-step explanation:

7(x + 9) + 5 = 96

Solve for x.

7(x + 9 ) + 5 = 96

Step 1 :- Distribute 7.

7 × x + 7 × 9 + 5 = 967x + 63 + 5 = 96

Step 2:- Add 63 and 5.

7x + 68 = 96

Step 3 :- Move constant to the right-hand side and change their sign.

7x = 96 - 68

Step 4 :- Subtract 68 from 96.

7x = 28

Step 5 :- Divide both side by 7.

[tex]\frac{7x}{7} \\ [/tex] = [tex]\frac{ 28} {7}\\[/tex] x = 4

A man is lying on the beach, flying a kite. He holds the end of the kite string at ground level and estimates the angle of elevation of the kite to be 65°. If the string is 350 ft long, how high is the kite above the ground? (Round your answer to the nearest foot.) ft

Answers

Answer: [tex]317.2\ ft[/tex]

Step-by-step explanation:

Given

Angle of elevation [tex]\theta=65^{\circ}[/tex]

Length of string [tex]L=350\ ft[/tex]

Suppose the height of kite from the ground level is h

from the figure, we can write

[tex]\Rightarrow \sin \theta=\dfrac{h}{L}\\\\\Rightarrow \sin 65^{\circ}=\dfrac{h}{350}\\\\\Rightarrow h=350\sin 65^{\circ}\\\Rightarrow h=317.2\ ft[/tex]

Solve an equation to find the missing angle

Answers

11.

[tex]6x = 30 \\ x = \frac{30}{6} \\ x = 5[/tex]

Missing angle:

[tex]6x \\ = 6 \times 5 \\ = 30[/tex]

_________________________________________

12.

[tex](4 + 5x) + (x + 2) = 180 \\ 6x + 6 = 180 \\ 6x = 180 - 6 \\ 6x = 174 \\ x = \frac{174}{6} \\ x = 29[/tex]

Missing angle 1:

[tex](4 + 5x) \\ = 4 + (5 \times 29) \\ = 4 + 145 \\ = 149[/tex]

Missing angle 2:

[tex]x + 2 \\ = 29 + 2 \\ = 31[/tex]

_________________________________________

13.

[tex]5x + (3x + 12) = 180 \\ 8x + 12 = 180 \\ 8x = 180 - 12 \\ 8x = 168 \\ x = \frac{168}{8} \\ x = 21[/tex]

Missing angle 1:

[tex]5x \\ = 5 \times 21 \\ = 105[/tex]

Missing angle 2:

[tex](3x + 12) \\ = (3 \times 21) + 12 \\ = 63 + 12 \\ = 75[/tex]

_________________________________________

14.

[tex]32 + (6x + 4) = 90 \\ 36 + 6x = 90 \\ 6x = 90 - 36 \\ 6x = 54 \\ x = \frac{54}{6} \\ x = 9[/tex]

Missing angle:

[tex](6x + 4) \\ = (6 \times 9) + 4 \\ = 54 + 4 \\ = 58[/tex]

_________________________________________

15.

[tex](2x + 1) + (x + 2) = 90 \\ 3x + 3 = 90 \\ 3x = 90 - 3 \\ 3x = 87 \\ x = \frac{87}{3} \\ x = 29[/tex]

Missing angle 1:

[tex](2x + 1 ) \\ = ( 2 \times 29) + 1 \\ = 58 + 1 \\ = 59[/tex]

Missing angle 2:

[tex]x + 2 \\ = 29 + 2 \\ = 31[/tex]

_________________________________________

16.

[tex](3x + 1) + (4 + 2x) = 90 \\ 5x + 5 = 90 \\ 5x = 90 - 5 \\ 5x = 85 \\ x = \frac{85}{5} \\ x = 17[/tex]

Missing angle 1:

[tex](3x + 1) \\ = (3 \times 17) + 1 \\ = 51 + 1 \\ = 52[/tex]

Missing angle 2:

[tex](4 + 2x) \\ = 4 + (2 \times 17) \\ = 4 + 34 \\ = 38[/tex]

f(x) = 3x + 2
What is (5)?
O A. 21
B. 17
C. 15
O D. 10


Answers

B. 17

X=5

f(5) = 3x + 2
= 3(5) + 2
= 15+2
= 17

Hope this help you
Answer is 17...............

Plss Answer!!!!!!!!!​

Answers

Answer:

Ecosystem: Trees

Ways to protect are as follows:-

Control over Forest FireDon't waste paper. Plant a tree

pls help w explanation!!!

At her gym, Ximena spends 30 minutes on each aerobic workout and 20 minutes on each weight-lifting workout. Last week, Ximena spent between 190 and 230 minutes, inclusive, on 3 aerobic
workouts and w weight-lifting workouts. What is
one possible value of w?

Answers

Answer:

The possible values of W are 5, 6, and 7.

Step-by-step explanation:

Since at her gym, Ximena spends 30 minutes on each aerobic workout and 20 minutes on each weight-lifting workout, and last week, Ximena spent between 190 and 230 minutes, inclusive, on 3 aerobics workouts and W weight-lifting workouts, to determine what is one possible value of W the following calculation must be performed:

190 - (3 x 30) = X

190 - 90 = X

100 = X

100/2 = 5

140/2 = 7

Therefore, the possible values of W are 5, 6, and 7.

A new school has x day students and y boarding students.
The fees for a day student are $600 a term.
The fees for a boarding student are $1200 a term.
The school needs at least $720 000 a term.
Show that this information can be written as x + 2y ≥ 1200.​

Answers

Given:

The fees for a day student are $600 a term.

The fees for a boarding student are $1200 a term.

The school needs at least $720000 a term.

To show:

That the given information can be written as [tex]x + 2y\geq 1200[/tex].​

Solution:

Let x be the number of day students and y be the number of boarding students.

The fees for a day student are [tex]\$600[/tex] a term.

So, the fees for [tex]x[/tex] day students are [tex]\$600x[/tex] a term.

The fees for a boarding student are [tex]\$1200[/tex] a term.

The fees for [tex]y[/tex] boarding student are [tex]\$1200y[/tex] a term.

Total fees for [tex]x[/tex] day students and [tex]y[/tex] boarding student is:

[tex]\text{Total fees}=600x+1200y[/tex]

The school needs at least $720000 a term. It means, total fees must be greater than or equal to $720000.

[tex]600x+1200y\geq 720000[/tex]

[tex]600(x+2y)\geq 720000[/tex]

Divide both sides by 600.

[tex]\dfrac{600(x+2y)}{600}\geq \dfrac{720000}{600}[/tex]

[tex]x+2y\geq 1200[/tex]

Hence proved.

What number should be subtracted from -3/4 to get 5/6?

Answers

Answer:

Let that rational number to be subtracted be x.Given,-5/​6 - x = 4/9= - x = 4/9+5/6= - x = 23/18x = - 23/18.

Step-by-step explanation:

What is the value of x in the product of powers below?
6^9 x 6^x =6^2
0-11
O -7
07
11

Answers

Mmm, x is equals to -7....

The value of the x is -7.

What is an equation?

Two algebraic expressions having the same value and symbol '=' in between are called an equation.

We can use the rule for multiplying powers with the same base to simplify the left-hand side of the equation:

6⁹ × 6ˣ = 6⁽⁹⁺ˣ⁾

Now we can rewrite the equation as:

6⁽⁹⁺ˣ⁾ = 6²

Since the bases on both sides of the equation are the same, we can equate the exponents:

9 + x = 2

Subtracting 9 from both sides gives:

x = -7

Therefore, the value of x in the given equation is -7.

To learn more about the equation;

https://brainly.com/question/12788590

#SPJ7

f(b) = 7b^3 +8b and g(b) = b ^2+ b - 10. What is f(b)-g(b)?

Answers

Answer:

[tex]{ \bf{f(b) - g(b) : }} \\ { \tt{ = ( {7b}^{3} + 8b) - ( {b}^{2} + b - 10)}} \\ = ( {7b}^{3} - {b}^{2} + 7b + 10)[/tex]

Answer:

[tex]f(b) - g(b) = 7b^3 -b^2 + 7b +10[/tex]

Step-by-step explanation:

[tex]f(b) = 7b^3 + 8b \\\\g(b) = b^2 + b - 10\\\\f(b) - g(b) = (7b^3 + 8b ) - ( b^2 + b -10)[/tex]

               [tex]= 7b^3 + 8b - b^2 - b + 10\\\\=7b^3 - b^2 +7b + 10[/tex]

What is true about an equation with infinite solutions?
When both sides of the equation are simplified, the coefficients are the same.
When both sides of the equation are simplified, the constants are different.
There are no input values that will result in a true statement.
Only one input value will result in a true statement.

Answers

When both sides of the equation are simplified, the coefficients are the same.

Step-by-step explanation:

An equation has infinite solutions when both sides of the equation are simplified, the coefficients are the same

Answer:

The answer is A.

Help me with this question and do not answer unless you know the answer pls and thank you >:(

Answers

Answer:

1.5 * 1.5 = 2.25 sq miles

Step-by-step explanation:

1.5 * 1.5 = 2.25 sq miles

Plsss can someone answer the bottom question

Answers

Answer:

30

Step-by-step explanation:

[tex]\frac{75}{100} = \frac{x}{40}\\\\Cross- multiply:\\\\\\75*40=100x[/tex]

3000 = 100x

x = 30

Another way:

75/100 is in reality 3/4

and 3/4 of 40 = 30

40/4 = 10

10 * 3 = 30

If my answer is incorrect, pls correct me!

If you like my answer and explanation, mark me as brainliest!

-Chetan K

The answer is 30

Because its looking for 75% of 40, you have to turn 75% into a decimal by moving the placement by two.
75 turn into .75
After you do that you multiply 40 by .75 and you get 30, which is your answer. :)

helppp!! I NEED HELP PLEASE

Answers

Given:

The table of values for the function f(x).

To find:

The values [tex]f^{-1}(f(3.14))[/tex] and [tex]f(f(-7))[/tex].

Solution:

From the given table, it is clear that the function f(x) is defined as:

[tex]f(x)=\{(-14,11),(-7,-12),(-12,-5),(9,1),(10,-2),(-2,13)\}[/tex]

We know that if (a,b) is in the function f(x), then (b,a) must be in the function [tex]f^{-1}(x)[/tex]. So, the inverse function is defined as:

[tex]f^{-1}(x)=\{(11,-14),(-12,-7),(-5,-12),(1,9),(-2,10),(13,-2)\}[/tex]

And,

[tex]f^{-1}(f(a))=f^{1}(b)[/tex]

[tex]f^{-1}(f(a))=a[/tex]              ...(i)

Using (i), we get

[tex]f^{-1}(f(3.14))=3.14[/tex]

Now,

[tex]f(f(-7))=f(-12)[/tex]

[tex]f(f(-7))=5[/tex]

Therefore, the required values are [tex]f^{-1}(f(3.14))=3.14[/tex] and [tex]f(f(-7))=5[/tex].


The line CD is defined by the points C(-2,1) and D(10,7).
Find the equation of the line CD.

Answers

Answer:

The equation of the line is; y = 0.5·x + 2

Step-by-step explanation:

The points that define the line CD = C(-2, 1) and D(10, 7)

The equation of the line can be presented in the form of the general equation of a straight line, y = m·x + c

Where;

m = The slope of the line = [tex]\dfrac{7 - 1}{10 - (-2)} = \dfrac{1}{2} = 0.5[/tex]

c = The y-intercept

From the obtained slope, m = 0.5, using point D(10, 7), the equation of the line in point and slope form is therefore;

y - 7 = 0.5·(x - 10)

From the above equation of the line in point and slope form, we get the general form of the equation of the line as follows

y - 7 = 0.5·(x - 10) = 0.5·x - 5

y - 7 = 0.5·x - 5

y = 0.5·x - 5 + 7 = 0.5·x + 2

y = 0.5·x + 2

The equation of the straight line in general is y = 0.5·x + 2.

4.5hour into second

Answers

Step-by-step explanation:

4 .5 ×60min

4.5×60×60sec

16200 sec

1 hour -3600 sec                                                                                                    4,5 hoer - x sec                                                                                                         x=4,5* 3600=45*360=16200 sec                                                                     Answer: 4,5 hour = 16200 sec

How do I answer number 1

Answers

Answer:

#1 Haley is correct and Lacey is incorrect

#2 Kenji is incorrect.

Step-by-step explanation:

#1. x^3 (x^2) = x^5 but this same law doesn't apply to addition of numbers with exponents.

#2 The law of exponents doesn't apply to numbers with different bases that are not multiples of each other such as 3 and 4, so Kenji's simplification is not correct.

rationalization
[tex] \frac{6}{ \sqrt{3} } [/tex]

Answers

Answer:

2√3

Step-by-step explanation:

To rationalize, we multiply the denominator and numerator by the surd at the base

We have it that;

6/√3 * 1

= 6/√3 * √3/√3

= 6 √3/3 = 2√(3

Other Questions
Cul es el resultado de las estrategias en la lectura? Use the equation of the water level of the river represented by the equation y = 4x + 170, where x represents the number of years and y represents the total feet. What points are located on the line? Check all that apply. (170, 0) (0, 170) (12, 126) (50, 30) (5, 150) (60, 70) The slope of diagonal OA IS__, and its equation is__ I would appreciate the help on all of these :) Explain what the construction does and list the steps to creating the construction. Be as detailed as possible :D I need help with this physics question. Solve the given system by the substitution method. 3x + y = 14 7x - 4y = 20 What is the probality of rolling a 6-sided die on a getting a 1 or a prime number? How has our concept of rights changed or remained the same from the French Revolution And what type of immunity will the difference between word and react in the same manner regardless of the organism invading if 8km=5miles.how many miles are in 56m? A chemical bond formed when two atoms share two pairs of electrons is a ________ bond; it is best described as ________. Is the use of smartphones useful for the Skudents of secondary level? Write an e assay discussing the advantages md disadvantages of smartphones in about 300 words. HELP PLS ITS ALGEBRA 1 If the domain of a function that is reflected over the x-axis is (1, 5), (2, 1), (-1, -7), what is the range?A. (1, -5), (2, -1), (-1, 7)B. (5, 1), (1, 2), (-7, -1)C. (-5, -1), (-1, -2), (7, 1)D. (-1, 5), (-2, 1), (1, -7) Find the limit of f as or show that the limit does not exist. Consider converting the function to polar coordinates to make finding the limit easier. f(x,y) Help answering statement After simplification, the value of 1-2/1(1+2)-3/(1+2)(1+2+3)-4/(1+2+3)(1+2+3+4)-...-100/(1+2+...+99)(1+2+...+100) is a proper fraction in its lowest form. Find the difference of its numerator and denominator. On a Venn diagram, shade the region:1. AnB2. (BnC)nA Having some difficulty with this test, can you please help?